Proof of Outer Regularity of Lebesgue Measure on $mathbb{R}$












3












$begingroup$



Let $E subseteq mathbb{R}$ be a measurable set, and $varepsilon > 0 $. Show that there exists an open set $G supset E$ such that $mu(G setminus E) < epsilon$.




By the definition of Lebesgue measure we can find a countable collection of open intervals $(A_{n})_{n in mathbb{N}}$ such that
$$
sumlimits_{n=1}^{infty} |A_n| leq mu(E) + epsilon.
$$



And I think setting $G = cup_{n in mathbb{N}}A_n$ gives us what we want. But what about if $E$ is of infinite measure?










share|cite|improve this question











$endgroup$












  • $begingroup$
    Try consider the family $B_N=[-N,N]$. This is a countable collection of measurable sets having finite measure and that cover $mathbb{R}$. Approximate $E_N=Ecap B_N$ with an open set $G_N$ and then take the union.
    $endgroup$
    – Giuseppe Negro
    Oct 23 '14 at 16:04
















3












$begingroup$



Let $E subseteq mathbb{R}$ be a measurable set, and $varepsilon > 0 $. Show that there exists an open set $G supset E$ such that $mu(G setminus E) < epsilon$.




By the definition of Lebesgue measure we can find a countable collection of open intervals $(A_{n})_{n in mathbb{N}}$ such that
$$
sumlimits_{n=1}^{infty} |A_n| leq mu(E) + epsilon.
$$



And I think setting $G = cup_{n in mathbb{N}}A_n$ gives us what we want. But what about if $E$ is of infinite measure?










share|cite|improve this question











$endgroup$












  • $begingroup$
    Try consider the family $B_N=[-N,N]$. This is a countable collection of measurable sets having finite measure and that cover $mathbb{R}$. Approximate $E_N=Ecap B_N$ with an open set $G_N$ and then take the union.
    $endgroup$
    – Giuseppe Negro
    Oct 23 '14 at 16:04














3












3








3


1



$begingroup$



Let $E subseteq mathbb{R}$ be a measurable set, and $varepsilon > 0 $. Show that there exists an open set $G supset E$ such that $mu(G setminus E) < epsilon$.




By the definition of Lebesgue measure we can find a countable collection of open intervals $(A_{n})_{n in mathbb{N}}$ such that
$$
sumlimits_{n=1}^{infty} |A_n| leq mu(E) + epsilon.
$$



And I think setting $G = cup_{n in mathbb{N}}A_n$ gives us what we want. But what about if $E$ is of infinite measure?










share|cite|improve this question











$endgroup$





Let $E subseteq mathbb{R}$ be a measurable set, and $varepsilon > 0 $. Show that there exists an open set $G supset E$ such that $mu(G setminus E) < epsilon$.




By the definition of Lebesgue measure we can find a countable collection of open intervals $(A_{n})_{n in mathbb{N}}$ such that
$$
sumlimits_{n=1}^{infty} |A_n| leq mu(E) + epsilon.
$$



And I think setting $G = cup_{n in mathbb{N}}A_n$ gives us what we want. But what about if $E$ is of infinite measure?







measure-theory lebesgue-measure






share|cite|improve this question















share|cite|improve this question













share|cite|improve this question




share|cite|improve this question








edited Jan 10 at 17:14









Viktor Glombik

8211527




8211527










asked Oct 23 '14 at 15:56









Tom OfferTom Offer

14712




14712












  • $begingroup$
    Try consider the family $B_N=[-N,N]$. This is a countable collection of measurable sets having finite measure and that cover $mathbb{R}$. Approximate $E_N=Ecap B_N$ with an open set $G_N$ and then take the union.
    $endgroup$
    – Giuseppe Negro
    Oct 23 '14 at 16:04


















  • $begingroup$
    Try consider the family $B_N=[-N,N]$. This is a countable collection of measurable sets having finite measure and that cover $mathbb{R}$. Approximate $E_N=Ecap B_N$ with an open set $G_N$ and then take the union.
    $endgroup$
    – Giuseppe Negro
    Oct 23 '14 at 16:04
















$begingroup$
Try consider the family $B_N=[-N,N]$. This is a countable collection of measurable sets having finite measure and that cover $mathbb{R}$. Approximate $E_N=Ecap B_N$ with an open set $G_N$ and then take the union.
$endgroup$
– Giuseppe Negro
Oct 23 '14 at 16:04




$begingroup$
Try consider the family $B_N=[-N,N]$. This is a countable collection of measurable sets having finite measure and that cover $mathbb{R}$. Approximate $E_N=Ecap B_N$ with an open set $G_N$ and then take the union.
$endgroup$
– Giuseppe Negro
Oct 23 '14 at 16:04










1 Answer
1






active

oldest

votes


















1












$begingroup$

The key element here is that $mathbb{R}$ is $sigma$-finite. The rest is nitty-gritty and a $sum_{n=1}^infty {1 over 2^n} = 1$ trick.



Suppose for each set $E$ of finite measure and each $epsilon >0$ you can find some open set $G$ containing $E$ with $mu(G setminus E) < varepsilon$.



Now let $E$ be and $varepsilon>0$ be arbitrary, and let $E_n = E cap (n, n+1]$. Now let choose open $G_n$ such that
$$
E_n subset G_n
qquad text{and}
qquad mu(G_n setminus E_n) < varepsilon {1 over 3cdot2^{|n|}}.
$$



Now let $G = bigcup_n G_n$, which is open.
By monotonicity, we have
$$G setminus E = bigcup_n G_n setminus E subset bigcup_n G_n setminus E_n
implies
mu(G setminus E) le sum_n mu (G_n setminus E_n ) < varepsilon.
$$



Elaboration:
$G setminus E = (bigcup_n G_n) setminus E = bigcup_n (G_n setminus E)$. Now, since $G_n setminus E subset G_n setminus E_n$, we have
$bigcup_n (G_n setminus E) subset bigcup_n (G_n setminus E_n)$.






share|cite|improve this answer











$endgroup$













  • $begingroup$
    Thanks! This looks good. However I'm not sure about the final line. We have $ cup_n G_n setminus E subset (cup_n G_n) setminus E_n$, but surely not necessarily $ cup_n G_n setminus E subset cup_n (G_n setminus E_n)$ which is what we need for the conclusion?
    $endgroup$
    – Tom Offer
    Oct 23 '14 at 21:11












  • $begingroup$
    I added an elaboration.
    $endgroup$
    – copper.hat
    Oct 23 '14 at 21:27










  • $begingroup$
    Oh yeah. Can Just take the set difference inside the union first... thanks! I had thought that we'd need to use some sort of covering of $mathbb{R}$ to do this for sets of infinite measure. The $E_n$ you suggest don't even cover $mathbb{R}^+$, but it seems to work. I didn't expect it to!
    $endgroup$
    – Tom Offer
    Oct 23 '14 at 21:32








  • 1




    $begingroup$
    Well $A setminus B = A cap B^c$, so $(cup_n A) setminus B = (cup_n A) cap B^c = cup_n (A cap B^c) = cap_n (A setminus B)$.
    $endgroup$
    – copper.hat
    Oct 23 '14 at 21:34










  • $begingroup$
    Why do you care about a cover of $mathbb{R}^+$??? All you need is a collection of disjoint sets $E_n$ such that the measures are bounded and the union is $E$. Since $mathbb{R} = cup_n (n,n+1]$, you know that $E = cup_n E_n$.
    $endgroup$
    – copper.hat
    Oct 23 '14 at 21:36













Your Answer





StackExchange.ifUsing("editor", function () {
return StackExchange.using("mathjaxEditing", function () {
StackExchange.MarkdownEditor.creationCallbacks.add(function (editor, postfix) {
StackExchange.mathjaxEditing.prepareWmdForMathJax(editor, postfix, [["$", "$"], ["\\(","\\)"]]);
});
});
}, "mathjax-editing");

StackExchange.ready(function() {
var channelOptions = {
tags: "".split(" "),
id: "69"
};
initTagRenderer("".split(" "), "".split(" "), channelOptions);

StackExchange.using("externalEditor", function() {
// Have to fire editor after snippets, if snippets enabled
if (StackExchange.settings.snippets.snippetsEnabled) {
StackExchange.using("snippets", function() {
createEditor();
});
}
else {
createEditor();
}
});

function createEditor() {
StackExchange.prepareEditor({
heartbeatType: 'answer',
autoActivateHeartbeat: false,
convertImagesToLinks: true,
noModals: true,
showLowRepImageUploadWarning: true,
reputationToPostImages: 10,
bindNavPrevention: true,
postfix: "",
imageUploader: {
brandingHtml: "Powered by u003ca class="icon-imgur-white" href="https://imgur.com/"u003eu003c/au003e",
contentPolicyHtml: "User contributions licensed under u003ca href="https://creativecommons.org/licenses/by-sa/3.0/"u003ecc by-sa 3.0 with attribution requiredu003c/au003e u003ca href="https://stackoverflow.com/legal/content-policy"u003e(content policy)u003c/au003e",
allowUrls: true
},
noCode: true, onDemand: true,
discardSelector: ".discard-answer"
,immediatelyShowMarkdownHelp:true
});


}
});














draft saved

draft discarded


















StackExchange.ready(
function () {
StackExchange.openid.initPostLogin('.new-post-login', 'https%3a%2f%2fmath.stackexchange.com%2fquestions%2f987647%2fproof-of-outer-regularity-of-lebesgue-measure-on-mathbbr%23new-answer', 'question_page');
}
);

Post as a guest















Required, but never shown

























1 Answer
1






active

oldest

votes








1 Answer
1






active

oldest

votes









active

oldest

votes






active

oldest

votes









1












$begingroup$

The key element here is that $mathbb{R}$ is $sigma$-finite. The rest is nitty-gritty and a $sum_{n=1}^infty {1 over 2^n} = 1$ trick.



Suppose for each set $E$ of finite measure and each $epsilon >0$ you can find some open set $G$ containing $E$ with $mu(G setminus E) < varepsilon$.



Now let $E$ be and $varepsilon>0$ be arbitrary, and let $E_n = E cap (n, n+1]$. Now let choose open $G_n$ such that
$$
E_n subset G_n
qquad text{and}
qquad mu(G_n setminus E_n) < varepsilon {1 over 3cdot2^{|n|}}.
$$



Now let $G = bigcup_n G_n$, which is open.
By monotonicity, we have
$$G setminus E = bigcup_n G_n setminus E subset bigcup_n G_n setminus E_n
implies
mu(G setminus E) le sum_n mu (G_n setminus E_n ) < varepsilon.
$$



Elaboration:
$G setminus E = (bigcup_n G_n) setminus E = bigcup_n (G_n setminus E)$. Now, since $G_n setminus E subset G_n setminus E_n$, we have
$bigcup_n (G_n setminus E) subset bigcup_n (G_n setminus E_n)$.






share|cite|improve this answer











$endgroup$













  • $begingroup$
    Thanks! This looks good. However I'm not sure about the final line. We have $ cup_n G_n setminus E subset (cup_n G_n) setminus E_n$, but surely not necessarily $ cup_n G_n setminus E subset cup_n (G_n setminus E_n)$ which is what we need for the conclusion?
    $endgroup$
    – Tom Offer
    Oct 23 '14 at 21:11












  • $begingroup$
    I added an elaboration.
    $endgroup$
    – copper.hat
    Oct 23 '14 at 21:27










  • $begingroup$
    Oh yeah. Can Just take the set difference inside the union first... thanks! I had thought that we'd need to use some sort of covering of $mathbb{R}$ to do this for sets of infinite measure. The $E_n$ you suggest don't even cover $mathbb{R}^+$, but it seems to work. I didn't expect it to!
    $endgroup$
    – Tom Offer
    Oct 23 '14 at 21:32








  • 1




    $begingroup$
    Well $A setminus B = A cap B^c$, so $(cup_n A) setminus B = (cup_n A) cap B^c = cup_n (A cap B^c) = cap_n (A setminus B)$.
    $endgroup$
    – copper.hat
    Oct 23 '14 at 21:34










  • $begingroup$
    Why do you care about a cover of $mathbb{R}^+$??? All you need is a collection of disjoint sets $E_n$ such that the measures are bounded and the union is $E$. Since $mathbb{R} = cup_n (n,n+1]$, you know that $E = cup_n E_n$.
    $endgroup$
    – copper.hat
    Oct 23 '14 at 21:36


















1












$begingroup$

The key element here is that $mathbb{R}$ is $sigma$-finite. The rest is nitty-gritty and a $sum_{n=1}^infty {1 over 2^n} = 1$ trick.



Suppose for each set $E$ of finite measure and each $epsilon >0$ you can find some open set $G$ containing $E$ with $mu(G setminus E) < varepsilon$.



Now let $E$ be and $varepsilon>0$ be arbitrary, and let $E_n = E cap (n, n+1]$. Now let choose open $G_n$ such that
$$
E_n subset G_n
qquad text{and}
qquad mu(G_n setminus E_n) < varepsilon {1 over 3cdot2^{|n|}}.
$$



Now let $G = bigcup_n G_n$, which is open.
By monotonicity, we have
$$G setminus E = bigcup_n G_n setminus E subset bigcup_n G_n setminus E_n
implies
mu(G setminus E) le sum_n mu (G_n setminus E_n ) < varepsilon.
$$



Elaboration:
$G setminus E = (bigcup_n G_n) setminus E = bigcup_n (G_n setminus E)$. Now, since $G_n setminus E subset G_n setminus E_n$, we have
$bigcup_n (G_n setminus E) subset bigcup_n (G_n setminus E_n)$.






share|cite|improve this answer











$endgroup$













  • $begingroup$
    Thanks! This looks good. However I'm not sure about the final line. We have $ cup_n G_n setminus E subset (cup_n G_n) setminus E_n$, but surely not necessarily $ cup_n G_n setminus E subset cup_n (G_n setminus E_n)$ which is what we need for the conclusion?
    $endgroup$
    – Tom Offer
    Oct 23 '14 at 21:11












  • $begingroup$
    I added an elaboration.
    $endgroup$
    – copper.hat
    Oct 23 '14 at 21:27










  • $begingroup$
    Oh yeah. Can Just take the set difference inside the union first... thanks! I had thought that we'd need to use some sort of covering of $mathbb{R}$ to do this for sets of infinite measure. The $E_n$ you suggest don't even cover $mathbb{R}^+$, but it seems to work. I didn't expect it to!
    $endgroup$
    – Tom Offer
    Oct 23 '14 at 21:32








  • 1




    $begingroup$
    Well $A setminus B = A cap B^c$, so $(cup_n A) setminus B = (cup_n A) cap B^c = cup_n (A cap B^c) = cap_n (A setminus B)$.
    $endgroup$
    – copper.hat
    Oct 23 '14 at 21:34










  • $begingroup$
    Why do you care about a cover of $mathbb{R}^+$??? All you need is a collection of disjoint sets $E_n$ such that the measures are bounded and the union is $E$. Since $mathbb{R} = cup_n (n,n+1]$, you know that $E = cup_n E_n$.
    $endgroup$
    – copper.hat
    Oct 23 '14 at 21:36
















1












1








1





$begingroup$

The key element here is that $mathbb{R}$ is $sigma$-finite. The rest is nitty-gritty and a $sum_{n=1}^infty {1 over 2^n} = 1$ trick.



Suppose for each set $E$ of finite measure and each $epsilon >0$ you can find some open set $G$ containing $E$ with $mu(G setminus E) < varepsilon$.



Now let $E$ be and $varepsilon>0$ be arbitrary, and let $E_n = E cap (n, n+1]$. Now let choose open $G_n$ such that
$$
E_n subset G_n
qquad text{and}
qquad mu(G_n setminus E_n) < varepsilon {1 over 3cdot2^{|n|}}.
$$



Now let $G = bigcup_n G_n$, which is open.
By monotonicity, we have
$$G setminus E = bigcup_n G_n setminus E subset bigcup_n G_n setminus E_n
implies
mu(G setminus E) le sum_n mu (G_n setminus E_n ) < varepsilon.
$$



Elaboration:
$G setminus E = (bigcup_n G_n) setminus E = bigcup_n (G_n setminus E)$. Now, since $G_n setminus E subset G_n setminus E_n$, we have
$bigcup_n (G_n setminus E) subset bigcup_n (G_n setminus E_n)$.






share|cite|improve this answer











$endgroup$



The key element here is that $mathbb{R}$ is $sigma$-finite. The rest is nitty-gritty and a $sum_{n=1}^infty {1 over 2^n} = 1$ trick.



Suppose for each set $E$ of finite measure and each $epsilon >0$ you can find some open set $G$ containing $E$ with $mu(G setminus E) < varepsilon$.



Now let $E$ be and $varepsilon>0$ be arbitrary, and let $E_n = E cap (n, n+1]$. Now let choose open $G_n$ such that
$$
E_n subset G_n
qquad text{and}
qquad mu(G_n setminus E_n) < varepsilon {1 over 3cdot2^{|n|}}.
$$



Now let $G = bigcup_n G_n$, which is open.
By monotonicity, we have
$$G setminus E = bigcup_n G_n setminus E subset bigcup_n G_n setminus E_n
implies
mu(G setminus E) le sum_n mu (G_n setminus E_n ) < varepsilon.
$$



Elaboration:
$G setminus E = (bigcup_n G_n) setminus E = bigcup_n (G_n setminus E)$. Now, since $G_n setminus E subset G_n setminus E_n$, we have
$bigcup_n (G_n setminus E) subset bigcup_n (G_n setminus E_n)$.







share|cite|improve this answer














share|cite|improve this answer



share|cite|improve this answer








edited Jan 10 at 16:24









Viktor Glombik

8211527




8211527










answered Oct 23 '14 at 16:16









copper.hatcopper.hat

127k559160




127k559160












  • $begingroup$
    Thanks! This looks good. However I'm not sure about the final line. We have $ cup_n G_n setminus E subset (cup_n G_n) setminus E_n$, but surely not necessarily $ cup_n G_n setminus E subset cup_n (G_n setminus E_n)$ which is what we need for the conclusion?
    $endgroup$
    – Tom Offer
    Oct 23 '14 at 21:11












  • $begingroup$
    I added an elaboration.
    $endgroup$
    – copper.hat
    Oct 23 '14 at 21:27










  • $begingroup$
    Oh yeah. Can Just take the set difference inside the union first... thanks! I had thought that we'd need to use some sort of covering of $mathbb{R}$ to do this for sets of infinite measure. The $E_n$ you suggest don't even cover $mathbb{R}^+$, but it seems to work. I didn't expect it to!
    $endgroup$
    – Tom Offer
    Oct 23 '14 at 21:32








  • 1




    $begingroup$
    Well $A setminus B = A cap B^c$, so $(cup_n A) setminus B = (cup_n A) cap B^c = cup_n (A cap B^c) = cap_n (A setminus B)$.
    $endgroup$
    – copper.hat
    Oct 23 '14 at 21:34










  • $begingroup$
    Why do you care about a cover of $mathbb{R}^+$??? All you need is a collection of disjoint sets $E_n$ such that the measures are bounded and the union is $E$. Since $mathbb{R} = cup_n (n,n+1]$, you know that $E = cup_n E_n$.
    $endgroup$
    – copper.hat
    Oct 23 '14 at 21:36




















  • $begingroup$
    Thanks! This looks good. However I'm not sure about the final line. We have $ cup_n G_n setminus E subset (cup_n G_n) setminus E_n$, but surely not necessarily $ cup_n G_n setminus E subset cup_n (G_n setminus E_n)$ which is what we need for the conclusion?
    $endgroup$
    – Tom Offer
    Oct 23 '14 at 21:11












  • $begingroup$
    I added an elaboration.
    $endgroup$
    – copper.hat
    Oct 23 '14 at 21:27










  • $begingroup$
    Oh yeah. Can Just take the set difference inside the union first... thanks! I had thought that we'd need to use some sort of covering of $mathbb{R}$ to do this for sets of infinite measure. The $E_n$ you suggest don't even cover $mathbb{R}^+$, but it seems to work. I didn't expect it to!
    $endgroup$
    – Tom Offer
    Oct 23 '14 at 21:32








  • 1




    $begingroup$
    Well $A setminus B = A cap B^c$, so $(cup_n A) setminus B = (cup_n A) cap B^c = cup_n (A cap B^c) = cap_n (A setminus B)$.
    $endgroup$
    – copper.hat
    Oct 23 '14 at 21:34










  • $begingroup$
    Why do you care about a cover of $mathbb{R}^+$??? All you need is a collection of disjoint sets $E_n$ such that the measures are bounded and the union is $E$. Since $mathbb{R} = cup_n (n,n+1]$, you know that $E = cup_n E_n$.
    $endgroup$
    – copper.hat
    Oct 23 '14 at 21:36


















$begingroup$
Thanks! This looks good. However I'm not sure about the final line. We have $ cup_n G_n setminus E subset (cup_n G_n) setminus E_n$, but surely not necessarily $ cup_n G_n setminus E subset cup_n (G_n setminus E_n)$ which is what we need for the conclusion?
$endgroup$
– Tom Offer
Oct 23 '14 at 21:11






$begingroup$
Thanks! This looks good. However I'm not sure about the final line. We have $ cup_n G_n setminus E subset (cup_n G_n) setminus E_n$, but surely not necessarily $ cup_n G_n setminus E subset cup_n (G_n setminus E_n)$ which is what we need for the conclusion?
$endgroup$
– Tom Offer
Oct 23 '14 at 21:11














$begingroup$
I added an elaboration.
$endgroup$
– copper.hat
Oct 23 '14 at 21:27




$begingroup$
I added an elaboration.
$endgroup$
– copper.hat
Oct 23 '14 at 21:27












$begingroup$
Oh yeah. Can Just take the set difference inside the union first... thanks! I had thought that we'd need to use some sort of covering of $mathbb{R}$ to do this for sets of infinite measure. The $E_n$ you suggest don't even cover $mathbb{R}^+$, but it seems to work. I didn't expect it to!
$endgroup$
– Tom Offer
Oct 23 '14 at 21:32






$begingroup$
Oh yeah. Can Just take the set difference inside the union first... thanks! I had thought that we'd need to use some sort of covering of $mathbb{R}$ to do this for sets of infinite measure. The $E_n$ you suggest don't even cover $mathbb{R}^+$, but it seems to work. I didn't expect it to!
$endgroup$
– Tom Offer
Oct 23 '14 at 21:32






1




1




$begingroup$
Well $A setminus B = A cap B^c$, so $(cup_n A) setminus B = (cup_n A) cap B^c = cup_n (A cap B^c) = cap_n (A setminus B)$.
$endgroup$
– copper.hat
Oct 23 '14 at 21:34




$begingroup$
Well $A setminus B = A cap B^c$, so $(cup_n A) setminus B = (cup_n A) cap B^c = cup_n (A cap B^c) = cap_n (A setminus B)$.
$endgroup$
– copper.hat
Oct 23 '14 at 21:34












$begingroup$
Why do you care about a cover of $mathbb{R}^+$??? All you need is a collection of disjoint sets $E_n$ such that the measures are bounded and the union is $E$. Since $mathbb{R} = cup_n (n,n+1]$, you know that $E = cup_n E_n$.
$endgroup$
– copper.hat
Oct 23 '14 at 21:36






$begingroup$
Why do you care about a cover of $mathbb{R}^+$??? All you need is a collection of disjoint sets $E_n$ such that the measures are bounded and the union is $E$. Since $mathbb{R} = cup_n (n,n+1]$, you know that $E = cup_n E_n$.
$endgroup$
– copper.hat
Oct 23 '14 at 21:36




















draft saved

draft discarded




















































Thanks for contributing an answer to Mathematics Stack Exchange!


  • Please be sure to answer the question. Provide details and share your research!

But avoid



  • Asking for help, clarification, or responding to other answers.

  • Making statements based on opinion; back them up with references or personal experience.


Use MathJax to format equations. MathJax reference.


To learn more, see our tips on writing great answers.




draft saved


draft discarded














StackExchange.ready(
function () {
StackExchange.openid.initPostLogin('.new-post-login', 'https%3a%2f%2fmath.stackexchange.com%2fquestions%2f987647%2fproof-of-outer-regularity-of-lebesgue-measure-on-mathbbr%23new-answer', 'question_page');
}
);

Post as a guest















Required, but never shown





















































Required, but never shown














Required, but never shown












Required, but never shown







Required, but never shown

































Required, but never shown














Required, but never shown












Required, but never shown







Required, but never shown







Popular posts from this blog

Can a sorcerer learn a 5th-level spell early by creating spell slots using the Font of Magic feature?

Does disintegrating a polymorphed enemy still kill it after the 2018 errata?

A Topological Invariant for $pi_3(U(n))$